Validez de la ley de Ampère en términos de HHH

Sabemos que el campo magnético auxiliar H es

H = 1 m 0 B METRO

y

× H = j F
pero esta ecuación diferencial generalmente no es válida en el límite de un cuerpo magnetizado debido a un cambio abrupto en la magnetización METRO en el límite.

De esta ecuación diferencial se sigue

H d yo = I F mi norte C
¿Es válida esta ecuación integral si se aplica a través de un límite de un material magnetizado (es decir, un extremo de la integral está dentro del cuerpo magnetizado y el otro está fuera)? ¿O hereda algún problema de su forma diferencial?

EDITAR: creo que mi pregunta no está clara, así que agregaré:

Derivamos la integral de bucle del rotacional utilizando el teorema de Stokes. Sin embargo, el rizo no está definido en el límite, ¿está bien definida la integral de bucle? Otra forma de decirlo es: ¿puedo usar el teorema de Stokes cuando el rizo de la función no está definido en alguna región? –

¿Por qué la ecuación curl H = j_f no sería válida en el límite? Creo que es.
La magnetización es discontinua, por lo que sus derivados tendrán una singularidad en mi humilde opinión.
Sí, la corriente es singular en el límite, pero estas ecuaciones siguen siendo válidas en el sentido de distribución.
Lo que me preocupa es cómo podemos aplicar el teorema de Stokes para una función discontinua. ¿Tiene alguna idea de dónde buscarlo, querido Jan Lalinsky? También lo pregunté en MSE, no hubo respuesta :( aquí math.stackexchange.com/questions/4034965/…
Puede probar algunos libros más antiguos sobre análisis vectorial, me viene a la mente AP Wills. La regla general es que si la expresión es lineal en un campo vectorial, las singularidades tipo delta no plantean ningún problema y todo, incluidos Gauss-Ostrogradskii y el teorema de Stokes, funciona. Simplemente no use la superficie/curva de integración que atraviesa la singularidad, eso no funciona bien: la singularidad tiene que estar dentro o fuera.
Entonces, ¿una integral de línea que corta (es decir, interseca la singularidad y continúa sin moverse a lo largo de las singularidades) no es un problema?
Depende del tipo de singularidad. La integral de la ley de Ampere para un camino que cruza un cuerpo magnetizado no tiene problema, H no tiene singularidad allí. Si la ruta integral de amperios pasa por una corriente de línea, entonces hay un problema: la integral es cero o I dependiendo de si la línea de corriente cruza una superficie definida por el bucle o no, por lo que no está claro qué valor asignar a la integral en caso de que la singularidad esté en el camino de integración.

Respuestas (4)

Si entiendo correctamente la pregunta, la respuesta se encuentra en el párrafo 6.3.3 del libro de Griffiths. Sí, hereda la discontinuidad. En este caso, aún podría usar la ecuación diferencial justo arriba y justo debajo de la discontinuidad, y tomar un camino de integración que cruce el límite le dará las condiciones de límite para el campo magnético y para el campo auxiliar. Otra referencia es esta:

https://unlcms.unl.edu/cas/physics/tsymbal/teaching/EM-913/section6-Magnetostatics.pdf

Publicado:

Sí, aún puedes usar el teorema, si eres cauteloso con los dominios de integración. La integral de bucle se definirá incluso en caso de discontinuidad por lo que dije en los comentarios.

¡Entonces la integral de bucle no es válida!
¿Qué quieres decir con no válido?
Me disculpo por mi quizás mala forma de plantear las cosas. Derivamos la integral de bucle del rotacional utilizando el teorema de Stokes. Sin embargo, el rizo no está definido en el límite, ¿está bien definida la integral de bucle? Otra forma de decirlo es: ¿puedo usar el teorema de Stokes cuando el rizo de la función no está definido en alguna región?
Sí, creo que se reduce a que la región de discontinuidad es un conjunto de medida nula, es decir, solo el límite entre las dos magnetizaciones y, dado que todo es finito (nada diverge), no contribuirá a la integral.
¿Alguna matemática que quieras agregar?
¿Qué es exactamente lo que necesita? H está bien definido en todas partes excepto en el límite. Cuando tomas su integral de contorno, puedes verlo como H multiplicado por una función escalón, entonces divides las integrales en dos partes, una arriba y otra más allá del límite y así todo queda bien definido
Independientemente del valor de H en el límite, la intersección entre el contorno sobre el que está integrando y el límite tiene una medida nula, ya que es solo un conjunto de dos puntos, por lo que la contribución a la integral también es nula.
Estoy de acuerdo en que uno puede escribir una integral de línea. ¿Podemos aplicar el teorema de Stokes incluso si tenemos una función discontinua?
He actualizado mi respuesta. En caso de que necesite más ayuda, debemos pasar al chat para evitar demasiados comentarios.

¿Es válida esta ecuación integral si se aplica a través de un límite de un material magnetizado (es decir, un extremo de la integral está dentro del cuerpo magnetizado y el otro está fuera)? ¿O hereda algún problema de su forma diferencial?

Hay una suposición con respecto a esta condición límite que a menudo se pasa por alto por varias razones, pero que a veces puede volverse problemática. Las limitaciones de esta suposición se analizan en detalle en las secciones I.5 y I.6 de Classical Electrodynamics, tercera edición de John D. Jackson (es decir, versión de portada azul). Así que empezamos con:

(0) C   H d yo = S   d a   [ j + t D ] norte
dónde H es el campo magnético (técnicamente, B es la inducción magnética), D es el desplazamiento eléctrico, j es la densidad de corriente (específicamente la densidad de corriente promedio macroscópica , vea las páginas 248-258 en el libro de Jackson para definición y derivación), S es una superficie cerrada con una unidad exterior normal norte , y t = t .

Por lo general, uno barre lo siguiente debajo de la alfombra, por así decirlo. Puede haber una densidad de corriente superficial, k , que existe en una capa delgada no más gruesa que la profundidad de la piel de un electrón en la superficie del material conductor, ya sea causada por campos variables en el tiempo o simplemente presente debido a alguna fuente. En tales escenarios, el lado derecho de la Ecuación 0 cambia a:

(1) S   d a   [ j + t D ] t = k t   Δ yo
dónde t es el vector unitario transversal a la superficie S y Δ yo es la longitud de escala del pastillero transversal a la superficie S .

Hay otro problema que surge en tal límite que involucra lo que Jackson llama densidades de carga superficiales verdaderamente microscópicas , es decir, ρ ( X ) = σ d ( X ) dónde σ es la densidad de carga superficial promedio y d ( X ) es la función delta de Dirac . El escenario idealizado que nos enseñan en clase es que σ existe en la superficie y que tiene espesor cero, es decir, no tiene densidad de carga en el interior de los conductores, sólo en la superficie. Sin embargo, la verdad es que ρ ( X ) se limita a dentro ± 2 angstroms de la ''superficie'' de la distribución iónica. Para casi todos los propósitos, existe una discontinuidad en el campo eléctrico en este límite, pero en realidad es probable que varíe en una longitud finita comparable a unos pocos anchos atómicos.

Sin embargo, el rizo no está definido en el límite, ¿está bien definida la integral de bucle? Otra forma de decirlo es: ¿puedo usar el teorema de Stokes cuando el rizo de la función no está definido en alguna región?

Depende de lo que está haciendo. ¿Está probando lo que Jackson llamaría lo microscópico o lo macroscópico ? Si es lo último, las cosas son mucho más fáciles y el lado derecho de la Ecuación 0 puede desaparecer si no hay una densidad de corriente superficial significativa, k , en el límite. Incluso si está presente, aún se puede trabajar con la Ecuación 0 o 1, según el escenario, y obtener resultados significativos para aproximaciones macroscópicas .

Nota al margen
No volví a derivar la definición de Jackson de microscópico frente a macroscópico porque tiene 10 páginas en su libro y no es realmente necesario aquí. Básicamente implica notar la diferencia entre los promedios de conjuntos espaciales y temporales y por qué los espaciales son la opción correcta, luego muchos detalles sobre por qué XYZ está bien bajo los límites de WUV. La distinción con respecto a esta pregunta es si el OP quiere modelar adecuadamente H a través del límite en escalas hasta la atómica o si están de acuerdo con las aproximaciones típicas a mayor escala de micrómetros y más (más o menos).

Estimada Honeste vivere, estoy estudiando su respuesta. Perdóname por no aceptar actualmente tu respuesta porque las matemáticas para mi nivel son más altas. Agradezco el tiempo que se tomó para escribir esta respuesta.
@YasirSadiq - No te preocupes. He repasado esas secciones con detalles sangrientos varias veces y todavía necesito volver a visitarlas de vez en cuando. Hay muchas cosas que barremos debajo de la alfombra , por así decirlo, sin darnos cuenta.
Estas cosas que no se mencionan lastiman a la mayoría de las personas como yo que no tienen a nadie a quien acudir para pedir ayuda.
@YasirSadiq: si ayuda, no se omiten durante la discusión con fines maliciosos. En general, los antecedentes necesarios para comprender las consecuencias de incluir tales efectos van más allá de los estudiantes principiantes. Sin embargo, siempre es una buena idea preguntar sobre los límites de varios teoremas, "leyes" y aproximaciones para obtener una comprensión intuitiva del problema. Mi respuesta es básicamente afirmar que la integral de contorno está perfectamente bien si no te importan las escalas y los procesos microfísicos. En matemáticas, es válido porque los supuestos imponen exactitud.
Hola, ¿se ha añadido la recompensa a tu cuenta?
@Kashmiri - Sí.

La integral de contorno a través del límite no debe considerarse como la aplicación directa del teorema de Stoke en esta área, sino con un espíritu similar al principio de continuación analítica.

El teorema de Stokes

C H × d yo = A × H d a = j f, adjunto .

Esta relación es aplicable cuando × H existe. Por supuesto, no se sostiene cuando × H diverge

En la región a través de la frontera, aunque × H diverge, la integral de área falló, pero la integral de contorno sigue funcionando. Luego adoptamos la integral de contorno como la definición, extendida a tales regiones.

Por lo tanto, la relación

C H × d yo = j f, adjunto .

es una continuación analítica en la región transfronteriza, incluso la × H diverge en el mismo.

Similarmente, mi diverge en r = 0 para 1 / r 2 campo, pero la integral de superficie sigue trabajando allí

mi d A = 1 ϵ 0

Por lo tanto, adoptamos el resultado de la integral de superficie para definir la fuerza de la divergencia.

r ^ r 2 = 4 π d ( r ) .

Gracias, ¿podría explicar por qué funciona la integral de contorno?
@YasirSadiq La integral está bien definida para una función continua por partes. Significa que Para cada punto, existe una región vecina finita, entonces la integral puede recibir una definición operativa.
@YasirSadiq: puede encontrar útil lo siguiente: math.stackexchange.com/q/780715/177342

Solo usa la ley de Ampère para B para obtener la corriente total. Luego reste la corriente ligada del resultado para obtener la corriente libre, si es necesario.

Es interesante ver que esto recibe votos negativos. Es quizás demasiado fácil y directo. ¿Qué tipo de fotones virtuales realmente transportan H?
Estimado my2cts, no lo voté negativo. Casi nunca voto negativo a nada. Me alegra que haya agregado algo, aunque no aborda directamente la pregunta.
Los habituales excelentes usuarios que votan negativo sin leer la respuesta. +1